inégalité de fonctions complexes

Prover que pour tout fonction $f(z)$ analytic dans $|z|

Réponses

  • Sorry,

    Prover que pour tout fonction $ f(z)$ analytic dans $ \vert z\vert
  • je dis surement des co**$^ies mais a vue de nez ca sent assez le principe du max (il y a des modules et inferieur au module sur le bord donc mon esprit limité conclut...)

    in english : try with the maximum principle ; good luck
  • Merci, mais je ne comprend pas précisément le corrige...

    in english: sorry for this ugly french... i thought about Maximum Modulus Principle, but i don't know how to apply it exactly.
  • Bonjour,

    Des idées en vrac:

    Poisson formula:
    $|f(x)|^{2}=\frac{1}{4\pi^{2}}|\int_{0}^{2\pi}f(e^{i\theta})\frac{1-x^{2}}{|e^{i\theta}-x|^{2}}d\theta|^{2}$
    Cauchy-Schwarz:
    $|f(x)|^{2} \leq \frac{1}{4\pi^{2}}\int_{0}^{2\pi}|f(e^{i\theta})|^{2}d\theta \int_{0}^{2\pi} |\frac{1-x^{2}}{|e^{i\theta}-x|^{2}}|^{2}d\theta$
    possibly Fubini to calculate or estimate $\int_{-1}^{1} \int_{0}^{2\pi} |\frac{1-x^{2}}{|e^{i\theta}-x|^{2}}|^{2}d\theta dx$.
    (It is a shame that we know that $\int_{0}^{2\pi} \frac{1-x^{2}}{|e^{i\theta}-x|^{2}}d\theta=1$ but that perhaps $|\frac{1-x^{2}}{|e^{i\theta}-x|^{2}}|^{2}$ is not $d\theta dx$ integrable)

    Remark: there is a similar result which says under the same assumptions:
    $|\int_{0}^{1}f(t)dt| \leq \frac{1}{2}\int_{0}^{2\pi}|f(e^{it})|dt$.

    Peut-être que j'aurai pu me passer d'essayer d'écrire en anglais pour ce que j'avais à dire...
  • Pardon, j'ai fait une erreur. Voici le bon message:

    Bonjour,

    Des idées en vrac:

    Poisson formula:
    $|f(x)|^{2}=\frac{1}{4\pi^{2}}|\int_{0}^{2\pi}f(e^{i\theta})\frac{1-x^{2}}{|e^{i\theta}-x|^{2}}d\theta|^{2}$
    Cauchy-Schwarz:
    $|f(x)|^{2} \leq \frac{1}{4\pi^{2}}\int_{0}^{2\pi}|f(e^{i\theta})|^{2}d\theta \int_{0}^{2\pi} |\frac{1-x^{2}}{|e^{i\theta}-x|^{2}}|^{2}d\theta$
    possibly Fubini to calculate or estimate $\int_{-1}^{1} \int_{0}^{2\pi} |\frac{1-x^{2}}{|e^{i\theta}-x|^{2}}|^{2}d\theta dx$.
    (It is a shame that we know that $\int_{0}^{2\pi} \frac{1-x^{2}}{|e^{i\theta}-x|^{2}}d\theta=2\pi$ but that perhaps $|\frac{1-x^{2}}{|e^{i\theta}-x|^{2}}|^{2}$ is not $d\theta dx$ integrable on $[0,2\pi] \times [-1,1]$)

    Remark: there is a similar result which says under the same assumptions:
    $|\int_{0}^{1}f(t)dt| \leq \frac{1}{2}\int_{0}^{2\pi}|f(e^{it})|dt$.

    Peut-être que j'aurai pu me passer d'essayer d'écrire en anglais pour ce que j'avais à dire...
  • Sinon, on peut utiliser tout simplement la formule de Cauchy ce qui amène suivant la même méthode à évaluer ou estimer l'intégrale: $\int_{-1}^{1} \int_{0}^{2\pi}\frac{1}{|e^{i\theta}-x|^{2}}d\theta dx$ sachant que $|e^{i\theta}-x|^{2}=1-2xcos(\theta)+x^{2}$.
  • Merci beacoup,

    mais comme $\int_{0}^{2\pi} \frac{1-x^2}{|e^{i \theta} - x|^2} d \theta = 2\pi $ implique que $\int_{0}{2\pi} \frac{1} {|e^{i \theta} - x|^2} d\theta = \frac{2\pi }{1-x^2}$ et $\frac{1}{1-x^2}$ n'est past integrable sur $[-1,1]$ de nouveau.
  • OK, il faut donc peut-être essayer avec le carré du noyau de Poisson (mais là encore, l'intégrabilité n'est peut-être même pas assurée). J'y réfléchirai plus tard car j'ai beaucoup à faire maintenant.

    Cordialement,

    F.F.
  • Bonsoir à tous,

    Je reviens donc au problème maintenant que j'ai du temps. Je pense que Yauhen a probablement obtenu la preuve de son inégalité entre temps mais peut-être que mon message intéressera les autres personnes
    qui se sont penchés sur la question.
    On a en fait l'inégalité plus précise:
    $\int_{-1}^{1}|f(x)|^{2}dx \leq \frac{1}{2}\int_{0}^{2\pi}|f(e^{i\theta})|^{2}d\theta$.
    Supposons cette inégalité démontrée pour $f$ à valeurs réelles sur l'axe réel. Montrons comment elle implique le cas général:
    Si $f(z)=\sum_{n \geq 0}u_{n}z^{n}$ est notre fonction de départ (analytique sur le disque unité et prolongeable par continuité sur le bord), avec $u_{n}=a_{n}+ib_{n}$ (où $a_{n},b_{n}$ sont réels), on remarque que l'hypothèse faite sur $f$ implique que
    $\sum_{n \geq 0}|u_{n}|^{2}=\sum_{n \geq 0}a_{n}^{2}+\sum_{n \geq 0}b_{n}^{2}$ est fini.
    En effet, d'une part, pour tout $r\in ]0,1[$,
    $\frac{1}{2\pi}\int_{0}^{2\pi}|f(re^{i\theta})|^{2}d\theta=\sum_{n\geq0}|u_{n}|^{2}r^{2n}$ et cette quantité tend (en croissant avec $r$) vers $\sum_{n \geq 0}|u_{n}|^{2}$ (Beppo-Levi).
    D'autre part, par uniforme continuité de $f(z)$ sur le disque unité fermé $\overline{D}$ (pas besoin de Lebesgue ici), on a que
    $\frac{1}{2\pi}\int_{0}^{2\pi}|f(re^{i\theta})|^{2}d\theta$ tend (en croissant avec $r$) vers
    $\frac{1}{2\pi}\int_{0}^{2\pi}|f(e^{i\theta})|^{2}d\theta$. Ainsi,
    $\frac{1}{2\pi}\int_{0}^{2\pi}|f(re^{i\theta})|^{2}d\theta$ est majoré pour tout $r \in ]0,1[$ par la quantité finie:
    $\frac{1}{2\pi}\int_{0}^{2\pi}|f(e^{i\theta})|^{2}d\theta$ égale à
    $\sum_{n \geq 0} |u_{n}|^{2}$.
    Si on note alors $g(z)=\sum_{n \geq 0}a_{n}z^{n}$
    et $h(z)=\sum_{n \geq 0}b_{n}z^{n}$, $h$ et $g$ sont continues sur
    $\overline{D}$ et analytiques à l'intérieur et ces fonctions vérifient de plus:
    $f=g+ih$. Par ailleurs, $a_{n}$ et $b_{n}$ étant pour tout $n$ des réels, on a aussi $\overline{g(z)}=g(\overline{z})$ et de même pour $h$ pour $z$ dans le disque unité. Ceci reste valable aussi sur le cercle unité grâce à des arguments de continuité. On remarque ensuite que
    $\frac{1}{2}\int_{0}^{2\pi}|f(e^{i\theta})|^{2}d\theta$ vaut
    $\frac{1}{2}\int_{0}^{2\pi}f(e^{i\theta})\overline{f(e^{i\theta})}d\theta$ lui même égal à
    $\frac{1}{2}\int_{0}^{2\pi}|g(e^{i\theta})|^{2}d\theta+
    \frac{1}{2}\int_{0}^{2\pi}|h(e^{i\theta})|^{2}d\theta+
    \frac{i}{2}\int_{0}^{2\pi}(-g(e^{i\theta})h(e^{-i\theta})+g(e^{-i\theta})h(e^{i\theta})d\theta$
    Mais cette dernière intégrale est nulle puisque l'intégrande est une fonction impaire de $\theta$. On en déduit finalement que:
    $\frac{1}{2}\int_{0}^{2\pi}|f(e^{i\theta})|^{2}d\theta=
    \frac{1}{2}\int_{0}^{2\pi}|g(e^{i\theta})|^{2}d\theta+
    \frac{1}{2}\int_{0}^{2\pi}|h(e^{i\theta})|^{2}d\theta$. En supposant donc l'inégalité démontrée pour les fonctions à valeurs réelles sur l'axe réel
    (fonctions telles que $g$ et $h$), on s'aperçoit que:
    $\int_{-1}^{1}|f(x)|^{2}dx=
    \int_{-1}^{1}|g(x)|^{2}dx+\int_{-1}^{1}|h(x)|^{2}dx \leq
    \frac{1}{2}\int_{0}^{2\pi}|g(e^{i\theta})|^{2}d\theta+\frac{1}{2}\int_{0}^{2\pi}|h(e^{i\theta})|^{2}d\theta=
    \frac{1}{2}\int_{0}^{2\pi}|f(e^{i\theta})|^{2}d\theta$
    On peut donc supposer $f$ à valeurs réelles sur l'axe réel (c'est en fait l'idée essentielle qu'il fallait avoir car alors $\int_{-1}^{1}|f(x)|^{2}dx=\int_{-1}^{1}f(x)^{2}dx$ et il est plus facile de travailler cette dernière expression en utilisant les outils que nous offre l'analyse complexe).
    Pour $r \in ]0,1[$, soit le contour $C_{r}$, frontière du demi-disque supérieur ${\ z, |z|0 \}$, parcouru dans le sens positif. Par le théorème de Cauchy,
    $\int_{C_{r}}f(z)^{2}dz=0$ mais l'intégrale de gauche vaut:
    $\int_{-r}^{r}f(x)^{2}dx+ir\int_{0}^{\pi}f(re^{i\theta})^{2}e^{i\theta}d\theta=0$
    D'où on déduit:
    $\int_{-r}^{r}f(x)^{2}dx \leq \int_{0}^{\pi}|f(re^{i\theta})|^{2}d\theta$. On montre de même l'inégalité suivante en raisonnant sur le demi-disque inférieur:
    $\int_{-r}^{r}f(x)^{2}dx \leq \int_{\pi}^{2\pi}|f(re^{i\theta})|^{2}d\theta$
    D'où, par addition:
    $2\int_{-r}^{r}f(x)^{2}dx \leq \int_{0}^{2\pi}|f(re^{i\theta})|^{2}d\theta$ et cette dernière quantité est inférieure à
    $\int_{0}^{2\pi}|f(e^{i\theta})|^{2}d\theta$ comme on l'a remarqué plus haut. Il suffit alors de faire tendre $r$ vers $1$ par valeurs inférieures dans
    $2\int_{-r}^{r}f(x)^{2}dx \leq \int_{0}^{2\pi}|f(e^{i\theta})|^{2}d\theta$ pour finalement conclure.

    Comme on peut le constater à la vue de cette démonstration, l'inégalité à établir (cas particulier de l'inégalité de Riesz-Fejer) reste valable pour les fonctions $f$ dans l'espace de Hardy $H^{2}$ et c'est naturellement dans ce cadre qu'elle est généralement énoncée.


    F.F.
  • Bonsoir à tous,

    Je reviens donc au problème maintenant que j'ai du temps. Je pense que Yauhen a probablement obtenu la preuve de son inégalité entre temps mais peut-être que mon message intéressera les autres personnes
    qui se sont penchés sur la question.
    On a en fait l'inégalité plus précise:
    $\int_{-1}^{1}|f(x)|^{2}dx \leq \frac{1}{2}\int_{0}^{2\pi}|f(e^{i\theta})|^{2}d\theta$.
    Supposons cette inégalité démontrée pour $f$ à valeurs réelles sur l'axe réel. Montrons comment elle implique le cas général:
    Si $f(z)=\sum_{n \geq 0}u_{n}z^{n}$ est notre fonction de départ (analytique sur le disque unité et prolongeable par continuité sur le bord), avec $u_{n}=a_{n}+ib_{n}$ (où $a_{n},b_{n}$ sont réels), on remarque que l'hypothèse faite sur $f$ implique que
    $\sum_{n \geq 0}|u_{n}|^{2}=\sum_{n \geq 0}a_{n}^{2}+\sum_{n \geq 0}b_{n}^{2}$ est fini.
    En effet, d'une part, pour tout
    $r\in ]0,1[$,
    $\frac{1}{2\pi}\int_{0}^{2\pi}|f(re^{i\theta})|^{2}d\theta=\sum_{n\geq0}|u_{n}|^{2}r^{2n}$ et cette quantité tend (en croissant avec $r$) vers $\sum_{n \geq 0}|u_{n}|^{2}$ par convergence monotone.
    D'autre part, par uniforme continuité de $f(z)$ sur le disque unité fermé $\overline{D}$ (pas besoin de Lebesgue), on a que
    $\frac{1}{2\pi}\int_{0}^{2\pi}|f(re^{i\theta})|^{2}d\theta$ tend (en croissant avec $r$) vers
    $\frac{1}{2\pi}\int_{0}^{2\pi}|f(e^{i\theta})|^{2}d\theta$. Ainsi,
    $\frac{1}{2\pi}\int_{0}^{2\pi}|f(re^{i\theta})|^{2}d\theta$ est majoré pour tout $r \in ]0,1[$ par la quantité finie:
    $\frac{1}{2\pi}\int_{0}^{2\pi}|f(e^{i\theta})|^{2}d\theta$ égale à
    $\sum_{n \geq 0} |u_{n}|^{2}$.
    Si on note alors $g(z)=\sum_{n \geq 0}a_{n}z^{n}$
    et $h(z)=\sum_{n \geq 0}b_{n}z^{n}$, $h$ et $g$ sont continues sur
    $\overline{D}$ et analytiques à l'intérieur et ces fonctions vérifient:
    $f=g+ih$. Par ailleurs, $a_{n}$ et $b_{n}$ étant pour tout $n$ des réels, on a aussi $\overline{g(z)}=g(\overline{z})$ et de même pour $h$ pour $z$ dans le disque unité. Ceci reste valable aussi sur le cercle unité grâce à des arguments de continuité. On remarque ensuite que
    $\frac{1}{2\pi}\int_{0}^{2\pi}|f(e^{i\theta})|^{2}d\theta$ vaut
    $\frac{1}{2\pi}\int_{0}^{2\pi}f(e^{i\theta})\overline{f(e^{i\theta})}d\theta$ lui même égal à
    $\frac{1}{2}\int_{0}^{2\pi}|g(e^{i\theta})|^{2}d\theta+
    \frac{1}{2}\int_{0}^{2\pi}|h(e^{i\theta})|^{2}d\theta+
    \frac{i}{2}\int_{0}^{2\pi}(-g(e^{i\theta})h(e^{-i\theta})+g(e^{-i\theta})h(e^{i\theta}))d\theta$
    Mais cette dernière intégrale est nulle puisque l'intégrande est une fonction impaire de $\theta$. On en déduit finalement que:
    $\frac{1}{2\pi}\int_{0}^{2\pi}|f(e^{i\theta})|^{2}d\theta=
    \frac{1}{2}\int_{0}^{2\pi}|g(e^{i\theta})|^{2}d\theta+
    \frac{1}{2}\int_{0}^{2\pi}|h(e^{i\theta})|^{2}d\theta$. En supposant donc l'inégalité démontrée pour les fonctions à valeurs réelles sur l'axe réel
    (fonctions telles que $g$ et $h$), on s'aperçoit que:
    $\int_{-1}^{1}|f(x)|^{2}dx=
    \int_{-1}^{1}|g(x)|^{2}dx+\int_{-1}^{1}|h(x)|^{2}dx \leq
    \frac{1}{2}\int_{0}^{2\pi}|g(e^{i\theta})|^{2}d\theta+\frac{1}{2}\int_{0}^{2\pi}|h(e^{i\theta})|^{2}d\theta=
    \frac{1}{2\pi}\int_{0}^{2\pi}|f(e^{i\theta})|^{2}d\theta$
    On peut donc supposer $f$ à valeurs réelles sur l'axe réel (c'est en fait l'idée essentielle qu'il fallait avoir car alors $\int_{-1}^{1}|f(x)|^{2}dx=\int_{-1}^{1}f(x)^{2}dx$ et il est plus facile de travailler avec cette dernière expression en utilisant les outils que nous offre l'analyse complexe).
    Pour $r \in ]0,1[$, soit le contour $C_{r}$, frontière du demi-disque supérieur $\{z, |z|0\}$, parcouru dans le sens positif. Par le théorème de Cauchy,
    $\int_{C_{r}}f(z)^{2}dz=0$ mais l'intégrale de gauche vaut:
    $\int_{-r}^{r}f(x)^{2}dx+ir\int_{0}^{\pi}f(re^{i\theta})^{2}e^{i\theta}d\theta=0$
    D'où on déduit:
    $\int_{-r}^{r}f(x)^{2}dx \leq \int_{0}^{\pi}|f(re^{i\theta})|^{2}d\theta$. On montre de même l'inégalité suivante en raisonnant sur le demi-disque inférieur:
    $\int_{-r}^{r}f(x)^{2}dx \leq \int_{\pi}^{2\pi}|f(re^{i\theta})|^{2}d\theta$
    D'où, par addition:
    $2\int_{-r}^{r}f(x)^{2}dx \leq \int_{0}^{2\pi}|f(re^{i\theta})|^{2}d\theta$ et cette dernière quantité est inférieure à
    $\int_{0}^{2\pi}|f(e^{i\theta})|^{2}d\theta$ comme on l'a remarqué plus haut. Il suffit alors de faire tendre $r$ vers $1$ par valeurs inférieures dans
    $\int_{-r}^{r}f(x)^{2}dx \leq \int_{\pi}^{2\pi}|f(e^{i\theta})|^{2}d\theta$.

    Comme on peut le constater à la vue de cette démonstration, l'inégalité à établir (cas particulier de l'inégalité de Riesz-Fejer) reste valable pour les fonctions $f$ dans l'espace de Hardy $H^{2}$ et c'est naturellement dans ce cadre qu'il faut l'énoncer.

    Bonne soirée,

    F.F.
  • (J'ai précisé ici quelques points. Vous pouvez supprimer mes deux messages précédents si vous voulez, merci)

    Bonsoir à tous,

    Je reviens donc au problème maintenant que j'ai du temps. Je pense que Yauhen a probablement obtenu la preuve de son inégalité entre temps mais peut-être que mon message intéressera les autres personnes
    qui se sont penchés sur la question.
    On a en fait l'inégalité plus précise:
    $\int_{-1}^{1}|f(x)|^{2}dx \leq \frac{1}{2}\int_{0}^{2\pi}|f(e^{i\theta})|^{2}d\theta$.
    Supposons cette inégalité démontrée pour $f$ à valeurs réelles sur l'axe réel. Montrons comment elle implique le cas général:
    Si $f(z)=\sum_{n \geq 0}u_{n}z^{n}$ est notre fonction de départ (analytique sur le disque unité et prolongeable par continuité sur le bord), avec $u_{n}=a_{n}+ib_{n}$ (où $a_{n},b_{n}$ sont réels), on remarque que l'hypothèse faite sur $f$ implique que
    $\sum_{n \geq 0}|u_{n}|^{2}=\sum_{n \geq 0}a_{n}^{2}+\sum_{n \geq 0}b_{n}^{2}$ est fini.
    En effet, d'une part, pour tout
    $r\in ]0,1[$,
    $\frac{1}{2\pi}\int_{0}^{2\pi}|f(re^{i\theta})|^{2}d\theta=\sum_{n\geq0}|u_{n}|^{2}r^{2n}$ et cette quantité tend (en croissant avec $r$) vers $\sum_{n \geq 0}|u_{n}|^{2}$ quand $r$ tend en croissant vers 1 (Beppo Levi).
    D'autre part, par uniforme continuité de $f(z)$ sur le disque unité fermé $\overline{D}$ (pas besoin de Lebesgue), on a que
    $\frac{1}{2\pi}\int_{0}^{2\pi}|f(re^{i\theta})|^{2}d\theta$ tend (en croissant avec $r$) vers
    $\frac{1}{2\pi}\int_{0}^{2\pi}|f(e^{i\theta})|^{2}d\theta$ quand $r$ tend en croissant vers $1$. Ainsi,
    $\frac{1}{2\pi}\int_{0}^{2\pi}|f(re^{i\theta})|^{2}d\theta$ est majoré pour tout $r \in ]0,1[$ par la quantité finie:
    $\frac{1}{2\pi}\int_{0}^{2\pi}|f(e^{i\theta})|^{2}d\theta$ égale à
    $\sum_{n \geq 0} |u_{n}|^{2}$.
    Si on note alors $g(z)=\sum_{n \geq 0}a_{n}z^{n}$
    et $h(z)=\sum_{n \geq 0}b_{n}z^{n}$, $h$ et $g$ sont continues sur
    $\overline{D}$ et analytiques à l'intérieur et ces fonctions vérifient:
    $f=g+ih$. Par ailleurs, $a_{n}$ et $b_{n}$ étant pour tout $n$ des réels, on a aussi $\overline{g(z)}=g(\overline{z})$ et de même pour $h$ pour $z$ dans le disque unité. Ceci reste valable aussi sur le cercle unité grâce à des arguments de continuité. On remarque ensuite que
    $\frac{1}{2\pi}\int_{0}^{2\pi}|f(e^{i\theta})|^{2}d\theta$ vaut
    $\frac{1}{2\pi}\int_{0}^{2\pi}f(e^{i\theta})\overline{f(e^{i\theta})}d\theta$ lui même égal à
    $\frac{1}{2}\int_{0}^{2\pi}|g(e^{i\theta})|^{2}d\theta+
    \frac{1}{2}\int_{0}^{2\pi}|h(e^{i\theta})|^{2}d\theta+
    \frac{i}{2}\int_{0}^{2\pi}(-g(e^{i\theta})h(e^{-i\theta})+g(e^{-i\theta})h(e^{i\theta}))d\theta$
    Mais cette dernière intégrale est nulle puisque l'intégrande est une fonction impaire de $\theta$. On en déduit finalement que:
    $\frac{1}{2\pi}\int_{0}^{2\pi}|f(e^{i\theta})|^{2}d\theta=
    \frac{1}{2}\int_{0}^{2\pi}|g(e^{i\theta})|^{2}d\theta+
    \frac{1}{2}\int_{0}^{2\pi}|h(e^{i\theta})|^{2}d\theta$. En supposant donc l'inégalité démontrée pour les fonctions à valeurs réelles sur l'axe réel
    (fonctions telles que $g$ et $h$), on s'aperçoit que:
    $\int_{-1}^{1}|f(x)|^{2}dx=
    \int_{-1}^{1}|g(x)|^{2}dx+\int_{-1}^{1}|h(x)|^{2}dx \leq
    \frac{1}{2}\int_{0}^{2\pi}|g(e^{i\theta})|^{2}d\theta+\frac{1}{2}\int_{0}^{2\pi}|h(e^{i\theta})|^{2}d\theta=
    \frac{1}{2\pi}\int_{0}^{2\pi}|f(e^{i\theta})|^{2}d\theta$
    On peut donc supposer $f$ à valeurs réelles sur l'axe réel (c'est en fait l'idée essentielle qu'il fallait avoir car alors $\int_{-1}^{1}|f(x)|^{2}dx=\int_{-1}^{1}f(x)^{2}dx$ et il est plus facile de travailler cette dernière expression en utilisant les outils que nous offre l'analyse complexe).
    Pour $r \in ]0,1[$, soit le contour $C_{r}$, frontière du demi-disque supérieur $\{z, |z|0 \}$, parcouru dans le sens positif. Par le théorème de Cauchy,
    $\int_{C_{r}}f(z)^{2}dz=0$ mais l'intégrale de gauche vaut:
    $\int_{-r}^{r}f(x)^{2}dx+ir\int_{0}^{\pi}f(re^{i\theta})^{2}e^{i\theta}d\theta=0$
    D'où on déduit:
    $\int_{-r}^{r}f(x)^{2}dx \leq \int_{0}^{\pi}|f(re^{i\theta})|^{2}d\theta$. On montre de même l'inégalité suivante en raisonnant sur le demi-disque inférieur:
    $\int_{-r}^{r}f(x)^{2}dx \leq \int_{\pi}^{2\pi}|f(re^{i\theta})|^{2}d\theta$
    D'où, par addition:
    $2\int_{-r}^{r}f(x)^{2}dx \leq \int_{0}^{2\pi}|f(re^{i\theta})|^{2}d\theta$ et cette dernière quantité est inférieure à
    $\int_{0}^{2\pi}|f(e^{i\theta})|^{2}d\theta$ comme on l'a remarqué plus haut. Il suffit alors de faire tendre $r$ vers $1$ par valeurs inférieures dans
    $\int_{-r}^{r}f(x)^{2}dx \leq \int_{\pi}^{2\pi}|f(e^{i\theta})|^{2}d\theta$.

    Comme on peut le constater à la vue de cette démonstration, l'inégalité à établir (cas particulier de l'inégalité de Riesz-Fejer) reste valable pour les fonctions $f$ dans l'espace de Hardy $H^{2}$ et c'est naturellement dans ce cadre qu'il faut l'énoncer.

    Bonne soirée,

    F.F.
  • (Encore une correction, je pense cette fois-ci que mon message est sous version finale, excusez-moi)

    Bonsoir à tous,

    Je reviens donc au problème maintenant que j'ai du temps. Je pense que Yauhen a probablement obtenu la preuve de son inégalité entre temps mais peut-être que mon message intéressera les autres personnes
    qui se sont penchés sur la question.
    On a en fait l'inégalité plus précise:
    $\int_{-1}^{1}|f(x)|^{2}dx \leq \frac{1}{2}\int_{0}^{2\pi}|f(e^{i\theta})|^{2}d\theta$.
    Supposons cette inégalité démontrée pour $f$ à valeurs réelles sur l'axe réel. Montrons comment elle implique le cas général:
    Si $f(z)=\sum_{n \geq 0}u_{n}z^{n}$ est notre fonction de départ (analytique sur le disque unité et prolongeable par continuité sur le bord), avec $u_{n}=a_{n}+ib_{n}$ (où $a_{n},b_{n}$ sont réels), on remarque que l'hypothèse faite sur $f$ implique que
    $\sum_{n \geq 0}|u_{n}|^{2}=\sum_{n \geq 0}a_{n}^{2}+\sum_{n \geq 0}b_{n}^{2}$ est fini.
    En effet, d'une part, pour tout
    $r\in ]0,1[$,
    $\frac{1}{2\pi}\int_{0}^{2\pi}|f(re^{i\theta})|^{2}d\theta=\sum_{n\geq0}|u_{n}|^{2}r^{2n}$ et cette quantité tend (en croissant avec $r$) vers $\sum_{n \geq 0}|u_{n}|^{2}$ quand $r$ tend en croissant vers $1$ (Beppo-Levi).
    D'autre part, par uniforme continuité de $f(z)$ sur le disque unité fermé $\overline{D}$ (pas besoin de Lebesgue), on a que
    $\frac{1}{2\pi}\int_{0}^{2\pi}|f(re^{i\theta})|^{2}d\theta$ tend (en croissant avec $r$) vers
    $\frac{1}{2\pi}\int_{0}^{2\pi}|f(e^{i\theta})|^{2}d\theta$ quand $r$ tend en croissant vers $1$. Ainsi,
    $\frac{1}{2\pi}\int_{0}^{2\pi}|f(re^{i\theta})|^{2}d\theta$ est majoré pour tout $r \in ]0,1[$ par la quantité finie:
    $\frac{1}{2\pi}\int_{0}^{2\pi}|f(e^{i\theta})|^{2}d\theta$ égale à
    $\sum_{n \geq 0} |u_{n}|^{2}$.
    Si on note alors $g(z)=\sum_{n \geq 0}a_{n}z^{n}$
    et $h(z)=\sum_{n \geq 0}b_{n}z^{n}$, $h$ et $g$ sont continues sur
    $\overline{D}$ et analytiques à l'intérieur et ces fonctions vérifient:
    $f=g+ih$. Par ailleurs, $a_{n}$ et $b_{n}$ étant pour tout $n$ des réels, on a aussi $\overline{g(z)}=g(\overline{z})$ et de même pour $h$ pour $z$ dans le disque unité. Ceci reste valable aussi sur le cercle unité grâce à des arguments de continuité. On remarque ensuite que
    $\frac{1}{2}\int_{0}^{2\pi}|f(e^{i\theta})|^{2}d\theta$ vaut
    $\frac{1}{2}\int_{0}^{2\pi}f(e^{i\theta})\overline{f(e^{i\theta})}d\theta$ lui même égal à
    $\frac{1}{2}\int_{0}^{2\pi}|g(e^{i\theta})|^{2}d\theta+
    \frac{1}{2}\int_{0}^{2\pi}|h(e^{i\theta})|^{2}d\theta+
    \frac{i}{2}\int_{0}^{2\pi}(-g(e^{i\theta})h(e^{-i\theta})+g(e^{-i\theta})h(e^{i\theta}))d\theta$
    Mais cette dernière intégrale est nulle puisque l'intégrande est une fonction impaire de $\theta$. On en déduit finalement que:
    $\frac{1}{2}\int_{0}^{2\pi}|f(e^{i\theta})|^{2}d\theta=
    \frac{1}{2}\int_{0}^{2\pi}|g(e^{i\theta})|^{2}d\theta+
    \frac{1}{2}\int_{0}^{2\pi}|h(e^{i\theta})|^{2}d\theta$. En supposant donc l'inégalité démontrée pour les fonctions à valeurs réelles sur l'axe réel
    (fonctions telles que $g$ et $h$), on s'aperçoit que:
    $\int_{-1}^{1}|f(x)|^{2}dx=
    \int_{-1}^{1}|g(x)|^{2}dx+\int_{-1}^{1}|h(x)|^{2}dx \leq
    \frac{1}{2}\int_{0}^{2\pi}|g(e^{i\theta})|^{2}d\theta+\frac{1}{2}\int_{0}^{2\pi}|h(e^{i\theta})|^{2}d\theta=
    \frac{1}{2}\int_{0}^{2\pi}|f(e^{i\theta})|^{2}d\theta$
    On peut donc supposer $f$ à valeurs réelles sur l'axe réel (c'est en fait l'idée essentielle qu'il fallait avoir car alors $\int_{-1}^{1}|f(x)|^{2}dx=\int_{-1}^{1}f(x)^{2}dx$ et il est plus facile de travailler avec cette dernière expression en utilisant les outils que nous offre l'analyse complexe).
    Pour $r \in ]0,1[$, soit le contour $C_{r}$, frontière du demi-disque supérieur $\{z, |z|0 \}$, parcouru dans le sens positif. Par le théorème de Cauchy,
    $\int_{C_{r}}f(z)^{2}dz=0$ mais l'intégrale de gauche vaut:
    $\int_{-r}^{r}f(x)^{2}dx+ir\int_{0}^{\pi}f(re^{i\theta})^{2}e^{i\theta}d\theta=0$
    D'où on déduit:
    $\int_{-r}^{r}f(x)^{2}dx \leq \int_{0}^{\pi}|f(re^{i\theta})|^{2}d\theta$. On montre de même l'inégalité suivante en raisonnant sur le demi-disque inférieur:
    $\int_{-r}^{r}f(x)^{2}dx \leq \int_{\pi}^{2\pi}|f(re^{i\theta})|^{2}d\theta$
    D'où, par addition:
    $2\int_{-r}^{r}f(x)^{2}dx \leq \int_{0}^{2\pi}|f(re^{i\theta})|^{2}d\theta$ et cette dernière quantité est inférieure à
    $\int_{0}^{2\pi}|f(e^{i\theta})|^{2}d\theta$ comme on l'a remarqué plus haut. Il suffit alors de faire tendre $r$ vers $1$ par valeurs inférieures dans
    $2\int_{-r}^{r}f(x)^{2}dx \leq \int_{0}^{2\pi}|f(e^{i\theta})|^{2}d\theta$.

    Comme on peut le constater à la vue de cette démonstration, l'inégalité à établir (cas particulier de l'inégalité de Riesz-Fejer) reste valable pour les fonctions $f$ dans l'espace de Hardy $H^{2}$ et c'est naturellement dans ce cadre qu'il faut l'énoncer.

    Bonne soirée,

    F.F.
Connectez-vous ou Inscrivez-vous pour répondre.